- PowerScore Staff
- Posts: 5972
- Joined: Mar 25, 2011
- Sat Apr 24, 2021 8:41 am
#88230
Complete Question Explanation
(The complete setup for this game can be found here: lsat/viewtopic.php?f=174&p=88228#p88228)
The correct answer choice is (B).
The question stem asks you to supply a piece of information that will result in only one viable solution. The key here is to be careful using the templates with this question, because it is easy to fall into the trap of thinking that if a template has only one unplaced variable, placing that variable will result in that solution. A safer approach is to consider each variable:
(The complete setup for this game can be found here: lsat/viewtopic.php?f=174&p=88228#p88228)
The correct answer choice is (B).
The question stem asks you to supply a piece of information that will result in only one viable solution. The key here is to be careful using the templates with this question, because it is easy to fall into the trap of thinking that if a template has only one unplaced variable, placing that variable will result in that solution. A safer approach is to consider each variable:
X: X is already placed by the first rule, and thus placing X will not result in a single solution. Hence, answer choice (D) can be eliminated.
Y: Y is part of the ZY block, and we have already shown that the block has two placement options, and neither option forces just one solution. Therefore, Y is not a likely candidate to be part of the correct answer.
Z: Z is part of the ZY block, and we have already shown that the block has two placement options, and neither option forces just one solution. Thus, answer choice (E) can be eliminated.
W: The only role that W plays is as the necessary condition in the final rule. Just as making the necessary condition occur does not result in any inference (don’t make a Mistaken Reversal!), placing W in this game does not result in a single solution. Thus, answer choice (C) can be eliminated.
T: T is a powerful variable but only when it signs with agency S. As a sufficient condition, when T signs with S, then W must sign with S. As X is already signed with F, the ZY block must then sign with P, resulting in just one solution. Thus, answer choice (B) is correct. The other answer choice involving T, answer choice (A), is incorrect because when T signs with F, nothing must occur as a result, and therefore a single solution does not result from T signing with F.
Dave Killoran
PowerScore Test Preparation
Follow me on X/Twitter at http://twitter.com/DaveKilloran
My LSAT Articles: http://blog.powerscore.com/lsat/author/dave-killoran
PowerScore Podcast: http://www.powerscore.com/lsat/podcast/
PowerScore Test Preparation
Follow me on X/Twitter at http://twitter.com/DaveKilloran
My LSAT Articles: http://blog.powerscore.com/lsat/author/dave-killoran
PowerScore Podcast: http://www.powerscore.com/lsat/podcast/